Question

Use pumping lemma to show that whether L ={ aib3i | i≥1000 and i≤4000} is non-regular...

Use pumping lemma to show that whether L ={ aib3i | i≥1000 and i≤4000} is non-regular or regular. Show your steps against each of the pumping lemma claims.

0 0
Add a comment Improve this question Transcribed image text
Answer #1

Find the solution to the above question below, do read all the steps provided carefully and in case of any comments do comment below. If found helpful please upvote this.
Date : Page: To prove a bi regulas lemma. i 21000 is: 4000 non-regulas using pumping And 1) we assume that Laab is regula so

y es in both case 2 y lies in part aaaa b b b .bbb 1000 rines g gggg tines I hire jo 3 aaaa..aa bbb-bb b to cotines 5998 ving

Add a comment
Know the answer?
Add Answer to:
Use pumping lemma to show that whether L ={ aib3i | i≥1000 and i≤4000} is non-regular...
Your Answer:

Post as a guest

Your Name:

What's your source?

Earn Coins

Coins can be redeemed for fabulous gifts.

Not the answer you're looking for? Ask your own homework help question. Our experts will answer your question WITHIN MINUTES for Free.
Similar Homework Help Questions
ADVERTISEMENT
Free Homework Help App
Download From Google Play
Scan Your Homework
to Get Instant Free Answers
Need Online Homework Help?
Ask a Question
Get Answers For Free
Most questions answered within 3 hours.
ADVERTISEMENT
ADVERTISEMENT
ADVERTISEMENT